Determine la razón de la siguiente progresión geométrica: 81,27,9,3,1,....

Answers

Answer 1

Answer:

BẠN BỊ ĐIÊN À

Step-by-step explanation:

CÚT


Related Questions

HELPPPPP ASP PLZZZZZ

Answers

Answer:

[tex](f-g)(x)[/tex]

[tex]f(x)-g(x)[/tex]

[tex]x^{2} -6x-27-x+9[/tex]

[tex]x^{2} -7x-18[/tex]

----------------------

[tex](f*g)(x)[/tex]

[tex]=f(x)g(x)[/tex]

[tex](x^{2} -6x-27)(x-9)[/tex]

[tex]=x^{3} -15x^{2}+27x+243[/tex]

----------------------

[tex]\frac{f}{g} (x)[/tex]

[tex]\frac{x^{2} -6x-27}{x-9}[/tex]

[tex]\frac{(x-9)(x+3)}{x-9}[/tex]

[tex]x+3[/tex]

-----------------------

[tex](f+g)(x)[/tex]

[tex]f(x)+g(x)[/tex]

[tex]=x^{2} -6x-27+x-9[/tex]

[tex]=x^{2} -5x-36[/tex]

------------------------

OAmalOHopeO

------------------------

Im new to this app!
And im looking for help!!
Please help ASAP!!!
Please!!!!

Answers

y=x²-10x-7

a>0 so we will be looking for minimum

x=-b/2a=10/2=5

y=25-50-7=-32

Answer: (5;32)

Find the equation of the linear function represented by the table below in slope-intercept form.

Answers

Answer:

y=-4x-5

Step-by-step explanation:

The slope of the line is - 4, the equation of line is y=-4x-5

John runs a computer software store. Yesterday he counted 140 people who walked by the store, 63 of whom came into the store. Of the 63, only 25 bought something in the store.
(a) Estimate the probability that a person who walks by the store will enter the store. (Round your answer to two decimal places.)
(b) Estimate the probability that a person who walks into the store will buy something. (Round your answer to two decimal places.)
(c) Estimate the probability that a person who walks by the store will come in and buy something. (Round your answer to two decimal places.)
(d) Estimate the probability that a person who comes into the store will buy nothing. (Round your answer to two decimal places.)

Answers

Answer:

.................

Step-by-step explanation:

............

a. 45%
b. 39.68%
c. 17.86%
d. 60.32%

look at the image for the question

Answers

It’s 16
You would multiply height (4) by the base (length(2) x width(2)=4) which is 16

1. A helicopter is at a position from two VORS (VHF Omnidirectional
Radio Range, an aircraft navigation system operating in the VHF band -
not covered in chapter) as in the diagram shown below. Given the angles
shown, find the third angle.
Helicopter
74.0°
66.0°
VOR
VOR

Answers

The position of the helicopter and the two VORs forms a triangle and the third angle formed by these three entities is 40 degrees

The diagram is not shown; however, the question can still be answered.

The given angles are:

[tex]\theta_1 = 74.0^o[/tex]

[tex]\theta_2 = 66.0^o[/tex]

Represent the third angle as [tex]\theta_3[/tex]

The helicopter and the 2 VORs form a triangle.

So, we make use of the following theorem to calculate the third angle

[tex]\theta_1 + \theta_2 + \theta_3= 180^o[/tex] ---- sum of angles in a triangle

Substitute known values

[tex]74.0^o + 66.0^o + \theta_3= 180^o[/tex]

[tex]140.0^o + \theta_3= 180^o[/tex]

Collect like terms

[tex]\theta_3= 180 -140.0^o[/tex]

[tex]\theta_3= 40^o[/tex]

Hence, the third angle is 40 degrees.

Learn more about angles in a triangle at:

https://brainly.com/question/14780489

Find the area of the figure. (Sides meet at right angles.)

Answers

Answer:

56

Step-by-step explanation:

A=(3*4)+(4*(4+3+4))=56

Find an upper bound for E(h) the error of the machine approximation of the two-point forward difference formula for the first derivative and then find the h corresponding to the minimum of E(h).

The two-point forward difference formula for f'(x) is:_________

Answers

Answer:

I doubt it is not going to be a great

Please Help!! Whoever helps and gets it correct gets Brainliest and 5 star rating!!

Answers

Answer:

the reasoning states that "all the numbers begin with a 7 or an 8"

however this is not accurate as they can be in different placements

which can make a big difference in the total estimate.

for example:

the number could've been an 8, or an 80

they both begin with an 8

however have totally different values and could have messed up the total estimated number.

hope this helps :D

Seven and one-half foot-pounds of work is required to compress a spring 2 inches from its natural length. Find the work required to compress the spring an additional 3 inch.

Answers

Answer:

Apply Hooke's Law to the integral application for work: W = int_a^b F dx , we get:

W = int_a^b kx dx

W = k * int_a^b x dx

Apply Power rule for integration: int x^n(dx) = x^(n+1)/(n+1)

W = k * x^(1+1)/(1+1)|_a^b

W = k * x^2/2|_a^b

 

From the given work: seven and one-half foot-pounds (7.5 ft-lbs) , note that the units has "ft" instead of inches.   To be consistent, apply the conversion factor: 12 inches = 1 foot then:

 

2 inches = 1/6 ft

 

1/2 or 0.5 inches =1/24 ft

To solve for k, we consider the initial condition of applying 7.5 ft-lbs to compress a spring  2 inches or 1/6 ft from its natural length. Compressing 1/6 ft of it natural length implies the boundary values: a=0 to b=1/6 ft.

Applying  W = k * x^2/2|_a^b , we get:

7.5= k * x^2/2|_0^(1/6)

Apply definite integral formula: F(x)|_a^b = F(b)-F(a) .

7.5 =k [(1/6)^2/2-(0)^2/2]

7.5 = k * [(1/36)/2 -0]

7.5= k *[1/72]

 

k =7.5*72

k =540

 

To solve for the work needed to compress the spring with additional 1/24 ft, we  plug-in: k =540 , a=1/6 , and b = 5/24 on W = k * x^2/2|_a^b .

Note that compressing "additional one-half inches" from its 2 inches compression is the same as to  compress a spring 2.5 inches or 5/24 ft from its natural length.

W= 540 * x^2/2|_((1/6))^((5/24))

W = 540 [ (5/24)^2/2-(1/6)^2/2 ]

W =540 [25/1152- 1/72 ]

W =540[1/128]

W=135/32 or 4.21875 ft-lbs

Step-by-step explanation:

Round 573.073 to the greatest place

Answers

Answer:

574

Step-by-step explanation:

To round a two-digit number to the nearest ten, simply increase it or decrease it to the nearest number that ends in 0: When a number ends in 1, 2, 3, or 4, bring it down; in other words, keep the tens digit the same and turn the ones digit into a 0

Hope this helps <3

if U>T, R>Q, S>T and T>R, which of the following is TRUE?
1. S>Q
2. U > S
3.U > R​
A. 1 only
B. 2 only
C. 1 and 2
D. 2 and 3

Answers

Answer:

C. 1 and 2

Step-by-step explantation:

First, i would order them as U>T, T>R, R>Q, S>T

we can rewrite them as

U>T>R>Q,

now adding S, we get U>S>T>R>Q,

so U>S

We can also rewrite all of them as inequalities:

U-T>0

T-R>0

R-Q>0

S-T>0

Add R-Q and T-R

(R-Q)+(T-R)>0

-Q+T>0

T>Q, but because S>T we can say S>Q

Paul can install a 300-square-foot hardwood floor in 18 hours. Matt can install the same floor in 22 hours. How long would it take Paul and Matt to install the floor working together?
4 hours
9.9 hours
13.2 hours
30 hours

Answers

Answer:

9.9 hours

Step-by-step explanation:

The formula to determine the time together is

1/a+1/b = 1/c  where a and b are the times alone and c is the time together

1/18 + 1/22 = 1/c

The least common multiply of the denominators is 198c

198c(1/18 + 1/22 = 1/c)

11c+ 9c = 198

20c = 198

Divide by 20

20c/20 =198/20

c =9.9

Answer:

B - 9.9 hrs

Step-by-step explanation:

took the test.

8.9 x 10^3 in standard notation

Answers

Answer:

that is n standard notation mah frand

8.9 × 10^3 being scientific notation of " 8900 "

[tex]\huge\text{Hey there!}[/tex]

[tex]\large\textsf{8.9}\times\large\textsf{10}^\mathsf{3}\\\\\mathsf{10^3}\\\mathsf{= 10\times10\times10}\\\mathsf{= 100\times10}\\\mathsf{= \bf 1,000}\\\\\large\textsf{8.9}\times\large\textsf{1,000}\\\\\large\textsf{= \bf 8,900}\\\\\\\boxed{\boxed{\huge\text{Answer: \boxed{\underline{\underline{\bf 8,900}}}}}}\huge\checkmark[/tex]

[tex]\huge\text{Good luck on your assignment \& enjoy your day!}[/tex]

~[tex]\boxed{\huge\text{}\boxed{\frak{Amphitrite1040:)}}}[/tex]

find the missing side of the triangle

Answers

Answer:

x = 34

Step-by-step explanation:

Pytago:

x[tex]30^{2} + 16^{2} = x^2\\x = \sqrt{30^2 + 16^2} \\x = 34[/tex]

Find an equation of the plane orthogonal to the line
(x,y,z)=(0,9,6)+t(7,−7,−6)

which passes through the point (9, 6, 0).

Give your answer in the form ax+by+cz=d (with a=7).

Answers

The given line is orthogonal to the plane you want to find, so the tangent vector of this line can be used as the normal vector for the plane.

The tangent vector for the line is

d/dt (⟨0, 9, 6⟩ + ⟨7, -7, -6⟩t ) = ⟨7, -7, -6⟩

Then the plane that passes through the origin with this as its normal vector has equation

x, y, z⟩ • ⟨7, -7, -6⟩ = 0

We want the plane to pass through the point (9, 6, 0), so we just translate every vector pointing to the plane itself by adding ⟨9, 6, 0⟩,

(⟨x, y, z⟩ - ⟨9, 6, 0⟩) • ⟨7, -7, -6⟩ = 0

Simplifying this expression and writing it standard form gives

x - 9, y - 6, z⟩ • ⟨7, -7, -6⟩ = 0

7 (x - 9) - 7 (y - 6) - 6z = 0

7x - 63 - 7y + 42 - 6z = 0

7x - 7y - 6z = 21

so that

a = 7, b = -7, c = -6, and d = 21

An equation of the plane orthogonal to the line 7x - 7y - 6z = 21.

The given line is orthogonal to the plane you want to find,

So the tangent vector of this line can be used as

The normal vector for the plane.

The tangent vector for the line is,

What is the tangent vector?

A tangent vector is a vector that is tangent to a curve or surface at a given point.

d/dt (⟨0, 9, 6⟩ + ⟨7, -7, -6⟩t ) = ⟨7, -7, -6⟩

Then the plane that passes through the origin with this as its normal vector has the equation

⟨x, y, z⟩ • ⟨7, -7, -6⟩ = 0

We want the plane to pass through the point (9, 6, 0), so we just

translate every vector pointing to the plane itself by adding ⟨9, 6, 0⟩,

(⟨x, y, z⟩ - ⟨9, 6, 0⟩) • ⟨7, -7, -6⟩ = 0

Simplifying this expression and writing it in standard form gives

⟨x - 9, y - 6, z⟩ • ⟨7, -7, -6⟩ = 0

7 (x - 9) - 7 (y - 6) - 6z = 0

7x - 63 - 7y + 42 - 6z = 0

7x - 7y - 6z = 21

So that, a = 7, b = -7, c = -6, and d = 21.

To learn more about the equation of plane visit:

https://brainly.com/question/1603217

State if the scenario involves a permutation or a combination. Then find the number of possibilities.

A team of 15 basketball players needs to choose two players to refill the water cooler.

Permutation/Combination:

Answer:

Answers

Answer:

Permutation ; 210 ways

Step-by-step explanation:

Permutation and combination methods refers to mathematical solution to finding the number of ways of making selection for a group of objects.

Usually, selection process whereby the order of selection does not matter are being treated using permutation, while those which takes the order of selection into cognizance are calculated using combination.

Here, selecting 2 players from 15 ; since order does not matter, we use permutation ;

Recall :

nPr = n! ÷ (n - r)!

Hence,

15P2 = 15! ÷ (15 - 2)!

15P2 = 15! ÷ 13!

15P2 = (15 * 14) = 210 ways

Reason Can you subtract a positive integer from a positive integer
and get a negive result? Explain your answer.

Answers

Answer:

No

Step-by-step explanation:

No matter the situation, when you multiply a negative by a negativeyou get a positive and a positive by a positive you get a positive. but if its two different like a negative and a positive then its NEGITIVE.

let's say you have 23 and you're multiplying by 2.

It's always increasing so it doesnt ever reach the negitive numbers.

Find the length of the arc.

A. 539π/12 km
B. 9π/3 km
C. 9π/2 km
D. 18π km

Answers

Answer:

b because it is I found out cus I took test

The length of the arc 9π/2 km.

The answer is option C.9π/2 km.

What is the arc of the circle?

The arc period of a circle can be calculated with the radius and relevant perspective using the arc period method.

  ⇒angle= arc/radius

     ⇒  135°=arc/6km

     ⇒ arc =135°*6km

     ⇒arc=135°*π/180° * 6km

    ⇒arc = 9π/2 km

Learn more about circle here:-https://brainly.com/question/24375372

#SPJ2

Find the slope of the line that goes through the
(2,6) and (-1, -6)

Answers

We can use the formula y2-y1/x2-x1 to get our slope. y2 and x2 are our second y and x coordinates, meanwhile y1 and x1 are our first y and x coordinates. -6-6/-1 -2 is -12/-3. -12/-3 is 4, the slope is 4.
the slope is 1/4 because you use the slope intercept formula

please solve the question ​

Answers

Answer:

[tex]g(-1) = -1[/tex]

[tex]g(0.75) = 0[/tex]

[tex]g(1)= 1[/tex]

Step-by-step explanation:

Given

See attachment

Solving (a): g(-1)

We make use of:

[tex]g(x) = -1[/tex]

Because: [tex]-1 \le x < 0[/tex] is true for x =-1

Hence:

[tex]g(-1) = -1[/tex]

Solving (b): g(0.75)

We make use of:

[tex]g(x) = 0[/tex]

Because: [tex]0 \le x < 1[/tex] is true for x =0.75

Hence:

[tex]g(0.75) = 0[/tex]

Solving (b): g(1)

We make use of:

[tex]g(x) = 1[/tex]

Because: [tex]1 \le x < 2[/tex] is true for x =1

Hence:

[tex]g(1)= 1[/tex]

Solve for x: 10/3 = x/(−5/2)

Answers

9514 1404 393

Answer:

  x = -25/3

Step-by-step explanation:

Multiply by the inverse of the coefficient of x. Reduce the fraction.

  (-5/2)(10/3) = (-5/2)(x/(-5/2))

  -50/6 = x = -25/3

Answer:

-25/3

Step-by-step explanation:

the other person is also correct. khan said so

Which ratio is equal to 27 : 81?

Answers

3:9 and if you reduce it again, 1:3

Answer:

1:3

Step-by-step explanation:

27 : 81

Divide each side by 27

27/27 : 81/27

1:3

Identify the transformed function that represents f(x) = ln x stretched vertically by a factor of 17, reflected across the x-axis, and shifted by 19 units left.
A. g(x) = −17ln (x + 19)
B. g(x) = 17ln (x − 19)
C. g(x) = 17ln (x + 19)
D. g(x) = −17ln (x − 19)

Answers

Answer:

b

Step-by-step explanation:

ANSWER. EXPLANATION. The given logarithmic function is. The transformation,. stretches the graph of y=f(x) vertically by a factor of c units ...

4 votes

ANSWER[tex]y = - 3 ln(x - 7) [/tex]EXPLANATIONThe given logarithmic function is [tex]f(x) = ln(x) [/tex]The transformation, [tex]y = - cf(x - k)[/tex]stretches

The length of a rectangle is twice its width. If the area of the rectangle is 72in², find its perimeter

Answers

Let breadth be x

Length=2x

[tex]\\ \sf\longmapsto Area=Length\times Breadth[/tex]

[tex]\\ \sf\longmapsto 72=2x(x)[/tex]

[tex]\\ \sf\longmapsto 2x^2=72[/tex]

[tex]\\ \sf\longmapsto x^2=\dfrac{72}{2}[/tex]

[tex]\\ \sf\longmapsto x^2=36[/tex]

[tex]\\ \sf\longmapsto x=\sqrt{36}[/tex]

[tex]\\ \sf\longmapsto x=6[/tex]

Length=6×2=12inBreadth=6in

[tex]\\ \sf\longmapsto Perimeter=2(L+B)[/tex]

[tex]\\ \sf\longmapsto Perimeter=2(12+6)[/tex]

[tex]\\ \sf\longmapsto Perimeter=2(18)[/tex]

[tex]\\ \sf\longmapsto Perimeter=36in[/tex]

Graph y=|x|+5, how does it compare to parent graph y=|x|

Answers

9514 1404 393

Answer:

  it is shifted 5 units upward

Step-by-step explanation:

The y-coordinate is a measure of the distance above the x-axis. When 5 is added to a y-coordinate, the point is shifted 5 units upward.

The function y = |x| +5 adds 5 units to the y-value of every point of the graph of y = |x|. The graph of y=|x|+5 is shifted 5 units upward from the parent graph.

In a class of 70 pupils, 36 like tasty time , 34 like ice-
cream, 6 like both tasty time }
draw a Venn diagram to show the data.
find how
many
like neither tasty time nor ice-cream

Answers

Step-by-step explanation:

I think this might be the correct answer

The number of pupils that like neither tasty-time nor ice cream is 6 if in a class of 70 pupils, 36 like tasty time, 34 like ice cream, 6 like both tasty times.

What is the Venn diagram?

It is defined as the diagram that shows a logical relation between sets.

The Venn diagram consists of circles to show the logical relation.

We have:

In a class of 70 pupils, 36 like tasty time, 34 like ice cream, 6 like both tasty time.

Total = 70 pupils

Number of like tasty time = 36

Number of like ice cream = 34

Number of like both = 6

Let x be the total number of pupils that like neither tasty-time nor ice cream

The number of pupils that like ice cream only =  34 - 6 = 28

The number of pupils that like tasty-time only = 36 - 6 = 30

From the Venn diagram:

28 + 30 + 6 + x = 70

x = 70 - 64

x = 6

Thus, the number of pupils that like neither tasty-time nor ice cream is 6 if in a class of 70 pupils, 36 like tasty time, 34 like ice cream, 6 like both tasty times.

Learn more about the Venn diagram here:

brainly.com/question/1024798

#SPJ2

if 25 liters of milk can make 8 tin of cheese how many liters of milk would b needed to make 11 tin of cheese? ​

Answers

Answer:

Step-by-step explanation:

34.375 Liters of milk make 11 tins of cheese. Assuming that milk is available in whole liters only, 35 Liters of milk are needed.

Find the length represented by x for each pair of similar triangles.
18cm, 9cm, and x
30cm, 15cm, and 25cm

Answers

Answer:

15 cm

Step-by-step explanation:

Since the traingles are similar, we can find the ratio between the side lengths, and it will be the same for each side.

We can use the side length 9 and 15 to find this ratio. 15/9=5/3. So, the ratio of a side length of the larger triangle to the smaller one is 5/3, so our equation becomes 5/3 = 25/x. Use any method you like to find that x=15.

Hope this helped,

~cloud

Ron has 22 coins with a total value of $ 1.85. The coins are nickels (5 cents) and dimes (10 cents). How many of each coin does he have?
Please help asap and thank you in advance!!

Answers

Answer:

18 dimes and 1 nickle

Step-by-step explanation:

This is because 18 time 10 is 180 and one of 5 add it and this is $1.85

Remeber a dollar is 100 cents

15 dimes, $1.50 and 7 nickels 35¢.


22 coins in total, $1.85 in total.

Hope this helps.
Other Questions
The graph of a line is shown below. What is the equation of the line, in slope-intercept form, that is parallel to this line and has a y-intercept of 1? Marcus White has just been promoted to a manager. To give him access to the files that he needs, you make his user account a member of the Managers group, which has access to a special shared folder. Later that afternoon, Marcus tells you that he is still unable to access the files reserved for the Managers group. What should you do Which statement explains how you could use coordinate geometry to prove that quadrilateral ABCD is a square What would be the name of this compound? rewrite the following sentence after making necessary correctionQ:-he left the place with bag and baggage the temperature of fridge compartment of a refrigerator is set at 8 C . the fridge compartment is set at -10C . what is difference between these temprature sittings Point-Slope Form of a Line Find the total surface area. rectangular prism 4 A. 700 m B. 3,040 m C. 135 m D. 1,308 m Marking brainliest If a job interview says tell me about yourself what do u say What is the answer when you evaluate m + p - p2 6; use m = 5 and p = 6 ? If enough experimental data supports a hypothesis, then it:O A. Is proven 100% true.OB. Is falsified.OC. Is accepted as true until prolan false.OD. Becomes an Observational Law PLEASE HELP!! Important test!! Option 1 isn't coming up so if you could try to solve others that would be great! what is the answer? I need help!! please and thank you solve the equation:|x+3| >2x-1 A 26-year-old G2P1 woman at 41 weeks gestation is brought in by ambulance. The emergency medical technician reports that a pelvic examination performed 20 minutes ago when the patient had a severe urge to push revealed that she was fully dilated and the fetal station was 2. Fetal heart tones were confirmed to be in the 150s, with no audible decelerations. When the patient is placed on the fetal monitor, the heart rate is noted to be in the 60s. The maternal heart rate is recorded as 100. Without pushing, the fetal scalp is visible at the introitus. A repeat pelvic exam shows that the infant is in the occiput anterior position. What is the most appropriate next step in the management of this patient Complete the pattern ___ 8,579 ____85.7 8.57____ Challenge: For a particular job, the Amax Employment Agency charges a fee that is equal to 15% of the firstmonth's pay. If the job pays X dollars annually, express the agency fee algebraically. what are the problems of various employment sectors? Please help, question attached. which early governing group of the united states adopted the articles of confedertion and ratified it before the end of the revolutionary warA. First continental congressB. Second continental congressC. Sons of liberty